Đến nội dung

JUV nội dung

Có 136 mục bởi JUV (Tìm giới hạn từ 24-05-2020)



Sắp theo                Sắp xếp  

#637256 $\frac{1}{f(x)}+\frac{1}{f(...

Đã gửi bởi JUV on 31-05-2016 - 22:20 trong Phương trình hàm

Đầu tiên thế $x=0$,$x=-1$ thì ta có $f(0)=f(-1)=2$. Giả sử hàm không phải hàm hằng trên đoạn $[-1;0]$, nếu tồn tại 1 số $a$ thoả mãn mãn $-1<a<0$ sao cho tất cả mọi số $b$ nằm trong khoảng $(a;0)$ đều có $f(b)$ khác 2 thì theo tính liên tục của hàm số thì với mọi $b$ nằm trong khoảng đó thì $f(b)$ hoặc lớn hơn $2$, hoặc nhỏ hơn $2$.Nếu $f(b)>2$, xét $x\in (a;0)$, khi chọn $x$ tiến đủ gần đến 0 sao cho $x^2+2x$ nằm trong khoảng $(a;0)$ thì lúc đó $f(x)>2$,$f(x^2+x)>2$ nên $\frac{1}{f(x)}+\frac{1}{f(x^{2}+2x)}<1$(vô lí).Nếu $f(b)<2$ thì lúc đó theo tính liên tục của hàm số thì tồn tại $c\in (a;0)$ sao cho $c<0$ và với mọi $b\in (c;0)$ thì $2>f(b)>0$.

Xét $x\in (c;0)$, chọn $x$ tiến đủ gần về $0$ sao cho $x^2+2x\in (c;0)$, lúc đó $0<f(x)<2$,$0<f(x^2+2x)<2$, vì vậy $\frac{1}{f(x)}+\frac{1}{f(x^{2}+2x)}>1$(vô lí). Vậy không tồn tại số $a$ thoả mãn. Vì vậy tồn tại $-1<d<0$ sao cho với mọi $b\in (d;0)$  thì $f(b)=2$. Vì $\frac{1}{f(x)}+\frac{1}{f(x^{2}+2x)}=1$ nên nếu với mọi $b\in (d;0)$ thì $f(b^2+2b)=2$ .Vậy $f(x)=2$ với mọi $x\in (d^2+2d;0)$ nếu $f(x)=2$ với mọi $x\in (d;0)$. Lặp lại quá trình đó vô hạn lần thì ta sẽ chứng minh được $f(b)=2$ với mọi $b\in (-1;0)$. Xét$x>0$, ta có thể chứng minh tương tự thì với mọi $x>0$ thì $f(x)=2$. Vậy $f(x)=2$ với $x\geq -1$. Lại có $\frac{1}{f(x)}+\frac{1}{f(x^{2}+2x)}=1$ và $x^2+2x\geq -1$ với mọi $x$ nên $\frac{1}{f(x^{2}+2x)}=2$ với mọi $x$. Vậy $f(x)=2$ là hàm cần tìm




#651013 $\frac{q}{m+n}.\binom{m+n}{...

Đã gửi bởi JUV on 23-08-2016 - 22:54 trong Tổ hợp và rời rạc

Mình không giỏi lắm về khoản khai triển những biểu thức phức tạp nên sẽ nói hướng làm và việc trình bày cụ thể sẽ dành cho bạn Bảo:

Tất nhiên $q\neq 0$, nếu $q=1$ thì bước đầu tiên là ta sẽ di chuyển sang phải (hay di chuyển theo vector $(1,0)$). Ta sẽ quy nạp theo $m+n$ với $p$,$q$ bất kì, có thể xét riêng các trường hợp $m+n=1,2,3$ để suy ra dpcm.Giả sử bài toán đúng với $m+n=k$, xét $m+n=k+1$. Nếu $q=0$ thì ta có bài toán đúng, nếu $q>0$ thì đầu tiên ta có $2$ cách di chuyển:

+)Di chuyển theo vector $(0,1)$, lúc này ta tịnh tiến đồ thị theo vector $(0,-1)$ thì điểm $P$ sẽ thành điểm $(m,n-1)$, đường thẳng $L$ thành đường thẳng $y=px+q-1$, cách đi từ gốc $O$ (điểm ta đến được lúc đầu trước khi dùng phép tịnh tiến) đến $P$ là $\frac{q-1}{m+n-1}\binom{m+n-1}{m}$ theo giả thiết quy nạp

+)Di chuyển theo vector $(1,0)$, tịnh tiến theo vector $(-1,0)$, điểm đang đến trở thành gốc $O$, $P$ thành $(m-1,n)$, $L$ thành $y=px+p+q$, cách đi từ $O$ đến $P$ là $\frac{p+q}{m+n-1}\binom{m+n-1}{m-1}$

Cộng $2$ giá trị trên lại, lưu ý rằng $n=pm+q$, ta sẽ có cách đi từ $O$ đến $(m,n)$, suy ra dpcm theo nguyên lí quy nạp




#660969 $\sum_{k=0}^{n}\frac{a_{k}...

Đã gửi bởi JUV on 07-11-2016 - 16:14 trong Tổ hợp và rời rạc

Câu a chính là bất đẳng thức Lubell- Yamamoto- Meshalkin, có thể tìm chứng minh trên mạng. Còn câu b thì ta có thể chứng minh trực tiếp từ KQ câu a: $1\geq \sum_{i=0}^{n}\frac{a_i}{\binom{n}{i}}\geq \left ( \sum_{i=0}^{n}a_i \right )\frac{1}{\binom{n}{\left \lfloor \frac{n}{2} \right \rfloor}}\Rightarrow \left | \Im \right |\leq \begin{pmatrix} n\\ \left \lfloor \frac{n}{2} \right \rfloor \end{pmatrix}$




#655385 $\text{CMR}$ $\mathcal{G}$...

Đã gửi bởi JUV on 24-09-2016 - 19:39 trong Tổ hợp và rời rạc

Nếu tồn tại $1$ đỉnh nối với ít hơn $5$ đỉnh thì sẽ có ít nhất $4$ đỉnh không nối với đỉnh đó, $4$ đỉnh đó sẽ đôi một nối với nhau.Nếu có $1$ đỉnh $A$ nối với ít nhất $6$ điểm $A_1,A_2,A_3,A_4,A_5,A_6$. Nếu một trong $6$ điểm đó nối với nhiều nhất $2$ trong $6$ điểm thì trong ít nhất $3$ điểm còn lại, các điểm đôi một nối với nhau nên $3$ điểm đó với điểm $A$ tạo $G4$. Nếu tồn tại $1$ điểm $B$ nối với ít nhất $3$ điểm trong $6$ điểm thì trong $3$ điểm đó tồn tại $1$ cạnh, từ đó có $G4$ là $A,B$ và $2$ điểm được nối.Nếu mỗi điểm trong $9$ điểm đó nối với đúng $5$ đỉnh thì tổng số cạnh là $\frac{5\times 9}{2}\notin \mathbb{N}$(vô lí) $\Rightarrow$ $(Q.E.D)$




#634140 $f(x+y)+f(x).f(y) = f(xy)+f(x)+f(y)$

Đã gửi bởi JUV on 19-05-2016 - 21:26 trong Phương trình hàm

Let $P(x,y)$ be the assertion of $f(x+y)+f(x)f(y)=f(xy)+f(x)+f(y)$ for all $x,y\in \mathbb{R}^+$

$P(x,y+z)$ give us $f(x+y+z)=f(x) + f(y + z) + f(xy + xz)-f(x)f(y + z)=f(x)+f(y)+f(z)+f(xy)+f(yz)+f(zx) + f(x)f(y)f(z)-f(x)f(y)-f(y)f(z)-f(z)f(x)+f(x^2yz)-f(xy)f(xz)-f(x)f(yz)$ for all $x,y,z\in \mathbb{R}^+$

Similarly, use this with $P(y,z+x)$ give us $f(x^2yz)-f(xy)f(xz)-f(x)f(yz) = f(xy^2z)-f(xy)f(yz)-f(y)f(xz)$ for all $x,y,z\in \mathbb{R}^+$

Set $y=1$ in that equation give us $f(x^2z)-f(x)f(xz)-f(x)f(z)=f(xz)-f(x)f(z)-f(1)f(xz)$ for all $x,z\in \mathbb{R}^+$

Which is $f(x^2z)=(1-f(1))f(xz)+f(x)f(xz)$ for all $x,z\in \mathbb{R}^+$, so $f(xy)=(1-f(1))f(y)+f(x)f(y)$ for all $x,y\in \mathbb{R}^+$

Since $f(xy)=(1-f(1))f(y)+f(x)f(y)=(1-f(1))f(x)+f(x)f(y)$ for all $x,y\in \mathbb{R}^+$ give us $f(x)=c$ constant or $f(1)=1$

If $f(x)=c$, we get $c+c^2=3c$, so $f(x)=2$ for all $x\in \mathbb{R}^+$

Otherwise, $f(1)=1$, so $f(xy)=f(x)f(y)$ for all $x,y\in \mathbb{R}^+$, so $f(x+y)=f(x)+f(y)$ for all $x,y\in \mathbb{R}^+$

Since $f:\mathbb{R}^+ \rightarrow \mathbb{R}^+$, so we get $f(x)=cx$ for constant $c$, check in $P(x,y)$ give $c=1$

So the answer is $f(x)=2$ for all $x\in \mathbb{R}^+$ and $f(x)=x$ for all $x\in \mathbb{R}^+$

Why?




#664542 $m\leq 2^{n-1}-1$

Đã gửi bởi JUV on 13-12-2016 - 16:04 trong Tổ hợp và rời rạc

Ta sẽ quy nạp theo $n$, dễ thấy $n=2$ đúng. Giả sử bài toán đúng với $n=k$. Xét $S= \left \{ 1;2;...;k+1 \right \}$. Xét các cặp $(A;S\setminus A)$ với $A\in S, \left | A \right |\geq 2$, nếu tồn tại $M,N$ trong $m$ tập thuộc cùng $1$ cặp thì xét $A_i$ $\forall i=\overline{1,m}$, luôn tồn tại $B_i\in M$ để $A_i\cap M=B_i$. $\forall T\subset M,T\neq \varnothing,T\neq M$, gọi $d_T$ là số các tập $A_i$ để $A_i\cap M=T$ và $A_i\neq T$. Nếu $\exists A_i,A_j: A_i\cap N=A_j\cap N\neq \varnothing, A_i\cap M=T,A_j\cap M=M\setminus T$ thì $3$ tập $(A_i,A_j,M)$ không thoả mãn đề bài. Vậy $A_i\cap N\neq A_j\cap N$ $\forall A_i,A_j:A_i\cap M=T=M\setminus (A_j\cap M)$ $(A_i\neq T,A_j\neq M\setminus T)$. $N$ có $2^{\left | N \right |}-1$ tập con khác rỗng nên $d_T+d_{M\setminus T}\leq 2^{\left | N \right |}-1$.Với $T= \varnothing$, $d_{\varnothing}\leq 2^{\left | N \right |-1}-1$ theo giả thuyết quy nạp, mặt khác với $A_i,A_j$ chứa $M$ và khác $M$ thì $(A_i\cap N)\cap (A_j\cap N)\neq \varnothing$, nếu không thì $(A_i,A_j,N)$ là $3$ tập không thoả mãn. Từ đó suy ra $d_M\leq 2^{\left | N \right |-1}$, vậy $d_M+d_{\varnothing}\leq 2^{\left | N \right |}-1$. Vậy $d_T+d_{M\setminus T}\leq 2{^\left | N \right |}-1\forall T\subset M$ .Gọi $p$ là số tập con $S$ trong $m$ tập mà là tập con của $M$, vì $\left | M \right |<k+1$ nên theo giả thiết quy nạp thì $p\leq 2^{\left | M \right |-1}-1$. Vậy $m=\frac{\sum_{T\subset M}(d_T+d_{M\setminus T})}{2}+p\leq (2^{\left | N \right |}-1)2^{\left | M \right |-1}+2^{\left | M \right |-1}-1=2^{\left | M \right |+\left | N \right |-1}-1= 2^k-1$. Nếu không tồn tại $M,N$ thuộc cùng $1$ cặp, lúc đó $m\leq 2^{n-1}$. Nếu $m=2^{n-1}$, mỗi cặp chỉ có $1$ tập con được chọn. Dễ thấy tất cả tập con có số phần tử bằng $k$ và $k+1$ được chọn, có thể dễ dàng chứng minh nếu tất cả tập con có $i+1$ phần tử được chọn thì tất cả tập con có $i$ phần tử cũng được chọn với $i>\frac{k}{2}$. Vậy tất cả các tập con có $>\frac{k}{2}$ phần tử đều được chọn, dễ suy ra điều vô lí. Vây $m\leq 2^{n-1}-1$ 




#654000 $n\in \mathbb{N}, n\geq 2$. Chứng minh rằn...

Đã gửi bởi JUV on 13-09-2016 - 12:38 trong Tổ hợp và rời rạc

Chứng minh quy nạp: Bài toán hiển nhiên đúng với $n=2$, giả sử bài toán đúng với $n=k$, xét $n=k+1$ và $2^k+1$ tập con trong tập $\left \{ 1;2;...;k+1 \right \}$ được chọn. Gọi $A$ là tập các tập con được chọn mà không chứa $k+1$, $B$ là tập các tập con chứa $k+1$. Vì $\left | A \right |+\left | B \right |=2^k+1$ nên $1$ trong $2$ tập $A$ và $B$ chứa ít nhất $2^{k-1}+1$ phần tử. Nếu là tập $A$ thì tồn tại $3$ tập con $M$, $N$, $P$ thuộc $A$ mà $M=N\cup P$ theo nguyên lí quy nạp (do $A$ là tập con của tập các tập con của $\left \{ 1;2;...;k \right \}$). Nếu tập $B$ chứa ít nhất $2^{k-1}+1$ phần tử thì bỏ $k+1$ trong mỗi phần tử của $B$ ,gọi tập các phần tử của $B$ mà bỏ đi $k+1$ là $C$ .Lập luận tương tự ta tìm được $3$ tập $M$,$N$,$P$ của $C$ thỏa mãn $M=N\cup P$, nên $M\cup \left \{ k+1 \right \}=(N\cup\left \{ k+1 \right \})\cup(P\cup \left \{ k+1 \right \})$.Vậy tồn tại $3$ tập của $B$ mà tập này bằng hợp $2$ tập còn lại. Vì vậy trong mọi trường hợp, ta có dpcm




#531950 $p^3-q^5=(p+q)^2$

Đã gửi bởi JUV on 05-11-2014 - 12:17 trong Số học

1.Giải phương trình nghiệm nguyên dương: $x^2+2x+4y^2=37$

2. TÌm nghiệm nguyên dương của phương trình : $x^2+y^2=2011^{1995^k+1}(10-z)$

3. TÌm tất cả các số nguyên tố thỏa mãn: $p^3-q^5=(p+q)^2$

Bài 1: 

$(x+1)^2+(2y)^2=38$

Từ đấy tự suy ra

Bài 3:

$p^3-q^5=(p+q)^2$=>$p^3-q^5=p^2+q^2+2pq$

=>$q^2(q^3+1)$=$p(p^2-p-2q)$

Vì p,q là SNT nên

$q^3+1$ chia hết $p$

$p^2-p-2q$ chia hết chp $q^2$

Từ đó cũng có $p-1$ chia hết cho $q$ , đặt $p-1=uq$

Có $upq+2q$ chia hết cho $q^2$ => $up-2$ chia hết cho $q$

 $p-1$ chia hết cho $q$=>$pu-u$ chia hết cho $q$

=>$u-2$ chia hết cho $q$

Có $q^3+1$ chia hết cho $p$

nếu $q+1$ chia hết cho $p$ thì $q+1 \ge p$

Lại có $p-1$ chia hết cho $q$ nên $p-1 \ge q$

=>$p \ge q+1$

=>$p=q+1$=>$p=3$,$q=2$(không thỏa mãn)

Nếu $q^2-q+1$ chia hết cho p thì $q(q-1) \ge p-1$

Mà $u-2$ chia hết cho $q$

Nếu $u-2>0$ thì $u-2 \ge q$=>$u \ge q+2$=> $p-1 \ge q(q+2)$

Mà $q(q-1) \ge p-1$ (vô lí)

Vây $u-2=0$=>$p-1=2q$

thay vào pt tìm đuợc $q=3$,$p=7$




#531987 $p^3-q^5=(p+q)^2$

Đã gửi bởi JUV on 05-11-2014 - 17:28 trong Số học

Bài 2

BTP: Nếu $p$ là 1 SNT dạng $4k+3$ thì $a^2+b^2$ chia hết cho $p$ khi và chỉ khi $a$,$b$ chia hết cho $p$ (tự CM)

2011 là SNT có dạng $4k+3$ 

$x^2+y^2=(2011^((1995^k+1)))(10-z)$

=>$x$,$y$ chia hết cho 2011=>$x^2$,$y^2$ chia hết cho $2011^2$

Đặt $x^2=u^2.2011^2$, $y^2=v^2.2011^2$, ta có

$u^2+v^2=(2011^(1995^k-1)))(10-z)$

Lập luận tương tự, cũng có $u^2$,$v^2$ chia hết cho $2011^2$

vì $1995^k+1$ là số chẵn nên sau $(1995^k+1)/2$ bước, ta có $x^2$,$y^2$ chia hết cho $2011^(1995^k+1))$

Đặt $x^2=a^2(2011^(1995^k+1)))$

$y^2=b^2(2011^(1995^k+1)))$

Ta có $a^2+b^2=10-z<10$

Giải ra, ta có (a;b;z)=(1;2;5);(2;1;5)

Vậy nghiệm của PT là

(x;y;z)=($2011^(1995^k+1))$;$2.2011^(1995^k+1))$;$5$);($2.2011^(1995^k+1))$;$2011^(1995^k+1))$;$5$)




#670226 $x^{2}+y^{2}=n$

Đã gửi bởi JUV on 28-01-2017 - 15:33 trong Số học

Tất cả các số $n$ thoả mãn $v_p(n)\vdots 2$ với mọi $p$ nguyên tố dạng $4k+3$. Lưu ý rằng mọi số nguyên tố có dạng $4k+1$ đều biểu diễn được dưới dạng $x^2+y^2$ và nếu $a,b$ biểu diễn được thì $ab$ cũng biểu diễn được ($(x^2+y^2)(m^2+n^2)=(xm+yn)^2+(xn-ym)^2$). Ngược lại, dễ chứng minh nếu $n=x^2+y^2$ chia hết cho $p$ với $p$ nguyên tố dạng $4k+3$ thì cả $x,y$ đều chia hết cho $p$, từ đó tổng bình phương chia hết cho $p^2$ nên $v_p(n)\vdots 2$.




#642308 18th ELMO (ELMO Lives Mostly Outside)

Đã gửi bởi JUV on 26-06-2016 - 18:57 trong Thi HSG Quốc gia và Quốc tế

Thấy thầy Hùng với bạn Bảo chém hình ác quá nên mình cũng giải 1 câu tổ hợp cho đủ bộ.

Bài 5: Gọi $n$ điểm trong tập $S$ là $A_1;A_2;...;A_n$, xét $n\vdots 2$, đặt $t=\frac{n}{2}$ đầu tiên ta xét các đường tròn đường kính $A_{i}A_{i+t}$ với $i$ chạy từ $1$ đến $t$. Tất cả các hình tròn này đều đôi một giao nhau nên màu của $2$ đường tròn bất kì trong $t$ hình tròn này khác nhau nên cần ít nhất $t$ màu để tô. Cách tô như sau: Gọi $t$ màu là $a_1;a_2;...;a_t$, đầu tiên với mọi $i$ chạy từ $1$ đến $t$, cung $A_{i}A_{i+t}$ được tô bởi màu $a_i$. Sau đó với mọi $k<t$, đường tròn đường kính $A_{i}A_{i+k}$ được tô cùng màu với đường tròn đường kính $A_{i}A_{i+t}$ với $i$ chạy từ $1$ đến $n-k$, với mọi $k>t$ thì đường tròn đường kính $A_{i}A_{i+k}$ được tô cùng màu với đường tròn đường kính $A_{i+k-t}A_{i+k}$. Dễ thấy cách tô này thoả mãn đề bài

Xét $n$ là số lẻ, đặt $n=2t+1$, gọi $a$ là số màu ít nhất có thể tô các đường tròn, ta có $2t<n<2t+2$, theo chứng minh trên thì $t\leq a\leq t+1$ (Vì một cách tô với $n$ điểm bằng $a$ màu thì có thể suy ra $1$ cách tô với $2t$ điểm với nhiều nhất $a$ màu, tương tự với cách tô $2t+2$ điểm thì suy ra $1$ cách tô với $n$ điểm bằng nhiều nhất $t+1$ màu ). Nếu $a=t$ thì lúc đó xét các đường tròn đường kính $A_{i}A_{i}$ với $i$ chạy từ $1$ đến $t$, $t$ đường tròn này đôi một cắt nhau nên chúng được tô bằng $t$ màu khác nhau lần lượt là $a_1;a_2;...;a_n$ và đường tròn đường kính $A_{t+1}A_{n}$ được tô màu $a_1$ vì nó cắt $t-1$ đường tròn kia ngoại trừ đường tròn đường kính $A_{1}A_{t+1}$. Ta xét tiếp các đường tròn $A_{i}A_{i+t+1}$ với $i$ chạy từ $1$ đến $t$, $t$ đường tròn này đôi một cắt nhau nên được tô bằng tất cả $t$ màu đã nêu trên. Vì vậy trong $t$ đường tròn đó có $1$ đường tròn tô màu $a_1$, tuy nhiên đường tròn này cắt $1$ trong $2$ đường tròn đường kính $A_{1}A_{t+1}$ và $A_{t}A{n}$ suy ra điều vô lí vì cả $3$ đường tròn tô cùng 1 màu. Vì vậy $a>t$, nên $a=t+1$. Vì có $1$ cách tô với $n+1$ điểm bằng $t+1$ màu nên từ đó có 1 cách tô với $n$ điểm với $t+1$ màu. Vì vậy số màu ít nhất trong mọi trường hợp là $\left [ \frac{n+1}{2} \right ]$

P/s: ELMO là viết tắt của từ gì hay là tên 1 con rối trong Sesame street vậy?




#665012 2012 ELMO Shortlist C1

Đã gửi bởi JUV on 18-12-2016 - 17:16 trong Tổ hợp và rời rạc

Ta sẽ chứng minh bằng quy nạp rằng $m=2n+1$ là giá trị nhỏ nhất cần tìm. Dễ thấy mệnh đề đúng với $n=2$, giả sử mệnh đề đúng với $n=k$. Xét dãy gồm $m$ số $a_1,a_2,...,a_m$ thoả mãn với mọi lớp có thể có độ dài $k+1$, tồn tại một dãy con của dãy đó có lớp như thế. Giả sử $a_1=a_2$, ta xét các lớp có dạng $(-1;i_1;i_2;...;i_k)$ với $i_t\in \left \{ -1;1 \right \}\forall t=\overline{1,k}$ và $a_{t_1};a_{t_2};...;a_{t_{k+2}}$ là dãy có lớp như thế với $t_1<t_2<...<t_{k+2}$. Có $a_{t_1}\neq a_{t_2}$ nên $t_2>2$. Cũng có dãy $a_{t_2};a_{t_3};...;a_{t_{k+2}}$ là một dãy con có lớp $(i_1;i_2;...;i_k)$ và dãy đó là một dãy con của dãy $a_3,a_4,...,a_m$ nên khi cho các số $i_t$ thay đổi tạo thành tất cả các lớp có thể có độ dài $k$ thì dãy $a_3,a_4,...,a_m$ luôn có một dãy con có lớp như thế. Dãy trên gồm $m-2$ số nên theo giả thiết quy nạp thì $m-2\geq 2k+1$ nên $m\geq 2k+3$. Nếu $a_1\neq a_2$ thì lập luận tương tự với lớp độ dài $k+1$ có dạng $(1;i_1;i_2;...;i_k)$ với $i_t\in \left \{ -1;1 \right \}\forall t=\overline{1,k}$, cũng có $m\geq 2k+3$. Vậy giả thiết quy nạp đúng, hay $m\geq 2n+1$. Có thể xét dãy $2n+1$ số sau: $1,0,1,0,1,0,...,0,1$($n+1$ số $1$ và $n$ số $0$ xếp xen kẽ).




#650097 60 thí sinh phải giải 3 bài toán. hai thí sinh bất kì luôn có ít nhất một bài...

Đã gửi bởi JUV on 17-08-2016 - 19:06 trong Tổ hợp và rời rạc

a/ Giả sử không ai làm bài $3$, nếu có $1$ người không làm được bài $2$ thì người đó chỉ làm được bài $1$, lúc đó tất cả mọi người đều làm được bài $1$ vì mọi người đều có ít nhất chung $1$ bài làm được với người đó

b/ Giả sử có $1$ người chỉ làm được $1$ bài thì tất cả mọi người đều làm được bài đó. Nếu tất cả mọi người đều làm được ít nhất $2$ bài thì tổng số bài làm ít nhất là $60\times2=120$ nên có ít nhất $1$ bài được ít nhất $120:3=40$ người làm được




#671607 [Trường Xuân toán học miền nam 2016] Vietnam TST 2016 MOCK Test 2

Đã gửi bởi JUV on 14-02-2017 - 13:57 trong Thi HSG cấp Tỉnh, Thành phố. Olympic 30-4. Đề thi và kiểm tra đội tuyển các cấp.

Đào mộ !

Bài 4:

Nếu $m,n$ cùng chia hết cho $3$ thì đáp số là $mn$

Nếu $m$ chia hết cho $3$ còn $n$ thì không, đáp số là $m(n-1)$(Nếu có $m(n-1)+1$ số $1$ thì trong $m$ hàng sẽ có $1$ hàng chứa $n$ số $1$, tổng $n$ số đó không chia hết cho $3$)

Nếu $m \equiv n\not\equiv 0 \pmod{3}$ thì đáp số là $mn-max\left \{ m;n \right \}$, lập luận tương tự trường hợp trên.

Nếu $m \equiv n+1\equiv 2 \pmod{3}$, ta chứng minh sẽ có ít nhất $\frac{2}{3}(m+n)$ ô không chứa số $1$. Gọi các ô không chứa số $1$ là ô lạ, dễ thấy rằng không có ô lạ nào đứng một mình, tức là không có ô lạ nào khác đứng cùng hàng hoặc cột với nó (Nếu không thì tổng trên cột và hàng chứa ô lạ đó sẽ hơn kém nhau $1$ nên không thể cùng chia hết cho $3$). Ta sẽ tô màu các ô của bảng theo cách sau: Đầu tiên chọn $1$ ô lạ, tô màu tất cả các ô chưa được tô màu nằm cùng hàng và cột với ô đó, sau đó chọn $1$ ô lạ khác chưa được chọn và tiếp tục. Dễ thấy mỗi hàng và cột phải có ít nhất $1$ ô lạ nên rút cuộc tất cả các ô trên bảng đều được tô, hay $m+n$ hàng và cột sẽ được tô(một hàng hoặc cột được tô khi tất cả các ô trên nó được tô). Khi chọn $1$ ô lạ bất kì thì tô được thêm nhiều nhất $2$ đường ($1$ đường tức là $1$ hàng hoặc cột). Nếu sau khi chọn $1$ ô lạ ta tộ được $2$ đường chứa nó thì sẽ có $1$ ô lạ khác nằm cùng hàng hoặc cột với nó chưa được chọn (Nếu được chọn trước rồi thì sau khi chọn ô lạ kia, ta chỉ có thể tô thêm được $1$ đường qua nó). Vậy có cách chọn để cứ chọn $2$ ô lạ liên tiếp, ta tô thêm được nhiều nhất $3$ đường $\Rightarrow$ Số ô lạ ít nhất là $\frac{2}{3}(m+n)$. Vậy số ô có chứa số $1$ ít nhất là $mn-\frac{2}{3}(m+n)$. Lưu ý rằng nếu $max\left \{ m;n \right \}>2min\left \{ m;n \right \}$ thì $max\left \{ m;n \right \}>\frac{2}{3}(m+n)$, lập luận tương tự các tường hợp trên thì số số $1$ nhiều nhất là $mn-max\left \{ m;n;\frac{2}{3}(m+n) \right \}$. Dấu "=" xảy ra khi:

Với $max\left \{ m;n \right \}\leq 2min\left \{ m;n \right \}$(giả sử $n>m$), chọn $\frac{2n-m}{3}$ hàng, mỗi hàng điền $2$ số $0$ sao cho không có $2$ số $0$ nào cùng cột. Trong $\frac{2m-n}{3}$ cột không có số $0$, điền vào mỗi cột $2$ số $1$ sao cho không có $2$ số $1$ hoặc $2$ số $0$ và$1$ cùng hàng.

Nếu $n>2m$, từ $1$ bảng $m\times 2m$ được điền, ta chọn thêm $n-2m$ cột và điền vào mỗi cột đó số $0$ vào ô ở dưới cùng và số $1$ vào các ô còn lại




#659215 Đề chọn đội tuyển học sinh giỏi quốc gia tỉnh Vĩnh Phúc (ngày 2) 2016-2017

Đã gửi bởi JUV on 24-10-2016 - 20:57 trong Thi HSG cấp Tỉnh, Thành phố. Olympic 30-4. Đề thi và kiểm tra đội tuyển các cấp.

Câu 5: Ta có $a_{i+1}-a_i\leq 1\Rightarrow (a_{i+1}-i-1)-(a_i-i)\leq 0$ $\forall \ i=\overline{1,2015}$. Gọi $i,j$ là $2$ số thoả mãn $a_i=i$,$a_j=j$ với $j>i$, có $0=a_i-i\geq a_{i+1}-i-1 \geq a_j-j=0$ và theo tính duy nhất của $i$ và $j$ thì $j=i+1$. Cũng theo tính duy nhất của $i$, $a_{i-1}>i+1$,$a_{i+2}<i$ và nếu tồn tại $t>i+1$ để $a_t>i$ thì vì$a_{i+2}<i$ nên sẽ tồn tại $p>i+1$ để $a_p<i$,$a_{p+1}>i$ và $a_{p+1}-a_p>1$(vô lí). Vậy các số $(a_{i+2},a_{i+3},...,a_{2016})$ là hoán vị của $(1,2,...,i-1)$$\Rightarrow$ $i-1=2015-i$ hay $i=1008$. Giờ ta sẽ tìm cách hoán vị các số $a_{1010}$ đến $a_{2016}$ thoả mãn đề bài. Gọi $s$ là số lớn nhất thoả mãn $a_{1010},a_{1011},...,a_s$ lập thành cấp số cộng công sai $1$. Nếu $a_s<1007$, lúc đó ta có $a_{s+1}<a_s$ và tồn tại $k>s$ thoả mãn $a_k>a_s$. Khi đó sẽ tồn tại $p>s$ thoả mãn $a_p<a_s$ và $a_{p+1}>a_s$ nên $a_{p+1}-a_p>1$. Vậy $a_s=1007$, tương tự nếu $s_1$ là số lớn nhất thoả mãn $a_{s+1},a_{s+2},...,a_{s_1}$ lập thành cấp số cộng công sai $1$ thì $a_{s_1}=a_{1010}-1$. Tương tự như thế thì dãy số $a_{1010},a_{1011},...,a_{2016}$ sẽ được tạo theo cách sau: Số hạng đầu tiên chọn ngẫu nhiên rồi các số tiếp theo sẽ chọn sao cho các số hạng tiếp theo sẽ lập thành cấp số cộng đến khi có $1$ số là số hạng lớn nhất chưa được chọn trong dãy; số hạng sau đó được chọn ngẫu nhiên trong các số còn lại và cứ tiếp tục như thế.Những cách chọn ngẫu nhiên trong dãy được gọi là cách chọn đặc biệt. Gọi $1$ nhóm $(i_1,i_2,...,i_t)$ được xác định rằng $i_1,i_2,...,i_t$ lần lượt là số đầu tiên, thứ $2$,...,thứ $t-1$,thứ $t$ là tất cả các số được dùng trong cách chọn đặc biệt để gắn cho một số số xác định trong dãy $a_{1010},a_{1011},...,a_{2016}$. Dễ thấy $i_1>i_2>...>i_t$ nên số cách chọn dãy đó là số cách chọn các tập con trong tập $\left \{ 1,2,...,1007 \right \}$ và bằng $2^{1007}$. Dễ thấy nếu chọn $1$ nhóm thì chỉ có $1$ dãy $a_{1010},a_{1011},...,a_{2016}$ được tạo thành nên số dãy thoả mãn là $2^{1007}$. Tương tự số cách chọn dãy $a_1,a_2,...,a_{1007}$ là $2^{1007}$. Vậy tổng số hoán vị thoả mãn là $2^{2014}$




#654650 Đề chọn đội tuyển Quốc Gia Hà Tĩnh 2016-2017 (2 ngày)

Đã gửi bởi JUV on 18-09-2016 - 14:41 trong Thi HSG cấp Tỉnh, Thành phố. Olympic 30-4. Đề thi và kiểm tra đội tuyển các cấp.

Câu 4: Ta sẽ chứng minh rằng $a_i=min\left \{ a_1;a_2;...;a_i \right \}$ hoặc $a_i=max\left \{ a_1;a_2;...;a_i \right \}$ ,$i=\overline{4,2016}$. Thật vậy vì $2\sum_{i=1}^{2016}a_i=2016.2017$ nên $2a_{2016} \equiv 2 \pmod{2015}$ nên $a_{2016}$ nhận giá trị là $1$ hoặc $2016$. Mệnh đề đúng với $i=2016$, giả sử mệnh đề đúng đến $i=k>4$. Ta thấy rằng $(a_1,a_2,...,a_{i-1})$ là $1$ hoán vị của $i-1$ số nguyên liên tiếp $a+1;...;a+i-1$.Nếu $i$ là số lẻ thì ta có $2\sum_{t=1}^{i-2}a_t =2\sum_{t=1}^{i-1}(a+t)-2a_{i-1} \vdots i-2$ nên $2a_{i-1} \equiv 2a+2 \pmod{i-2} \Rightarrow a_{i-1}\equiv a+1 \pmod{i-2}$ (do $i-2$ là số lẻ). Vậy $a_{i-1}$ nhận giá trị $a+1$ hoặc $a+i-1$. Nếu $i$ là số chẵn, lập luận tương tự ta có  $2a_{i-1} \equiv 2a+2 \pmod{i-2}$ nên $a_{i-1}$ nhận $1$ trong $3$ giá trị $a+1,a+i-1,a+1+\frac{i-2}{2}$. Nếu $a_{i-1}=a+1+\frac{i-2}{2}$ thì ta có $2\sum_{t=1}^{i-3}a_t =2\sum_{t=1}^{i-1}(a+t)-2a_{i-1}-2a_{i-2} \vdots i-3$ nên $2a_{i-2} \equiv 2a+i \pmod{i-3}$. Vậy $a_{i-3}=a+1+\frac{i-2}{2}=a_{i-2}$ (vô lí). Từ đó mệnh đề đúng với $i=k-1$. Mỗi cách chọn $(a_4;a_5;...;a_{2016})$ ứng với $1$ dãy nhị phân độ dài $2013$ với số thứ $i$ từ trái sang phải là $1$ nếu $a_{i+3}=max\left \{ a_1;a_2,...;a_{i+3} \right \}$, bằng $0$ trong trường hợp còn lại. Có tổng cộng $2^{2013}$ cách chọn dãy nhị phân nên có từng đấy cách chọn bộ số $(a_4;a_5;...;a_{2016})$. Với ba số $a_1;a_2;a_3$ có tổng cộng $6$ hoán vị nên tổng cộng số các hoán vị $2016$ số là $3.2^{2014}$




#652230 Đề kiểm tra Trường thu Toán học Hùng Vương 2016

Đã gửi bởi JUV on 01-09-2016 - 13:24 trong Thi HSG cấp Tỉnh, Thành phố. Olympic 30-4. Đề thi và kiểm tra đội tuyển các cấp.

Câu 4:

Kí hiệu $S=\left \{ 0;1;...;2017^{2016} \right \}$. Gọi $A_i=\left \{ 2^i3^j\mid j=\overline{0,2016} \right \}$ với mỗi $i\in S$. Gọi $f_n(A_i)$ là màu dùng để tô màu số lớn thứ $n$ trong tập $A_i$ với $n=\overline{1,2017}$, $i=\overline{0,2017^{2016}}$ Xét $g(A_i)=(f_1(A_i);f_2(A_i);...;f_{2017}(A_i))$. Vì có $2017^{2016}+1$ tập $A_i$ nên $\exists i,j\in S,i\neq j: g(A_i)=g(A_j)$. Vì $A_i$ có $2017$ phần từ nên $\exists m,n: 0\leq m<n\leq2016$: $2^{i}3^{m}$ và $2^{i}3^{n}$ tô cùng màu. Lại có $g(A_i)=g(A_j)$ nên $2^j3^m$ và $2^j3^n$ tô cùng màu với $2^i3^m$ và $2^i3^n$. Giả sử $i<j$, chọn $a=2^i3^m$,$b=2^j3^m$,$c=2^i3^n$,$d=2^j3^n$




#650628 Đề kiểm tra đội tuyển toán Chuyên Bảo Lộc (Lâm Đồng)

Đã gửi bởi JUV on 21-08-2016 - 13:03 trong Thi HSG cấp Tỉnh, Thành phố. Olympic 30-4. Đề thi và kiểm tra đội tuyển các cấp.

Câu 5(lần I):

a/ liệt kê tất cả các trường hợp, ta có các giá trị là $f_4=3,4$

b/ Nếu có $2n-3$ số thoả mãn thì có nghĩa tất cả các số từ $3$ đến $2n-1$ đều là tổng của $2$ số khác màu. Giả sử $n$ tô màu xanh, có $2n-1=n+n-1$ nên $n-1$ tô màu đỏ, có $2n-2=n+n-2$ nên $n-2$ cũng được tô đỏ, $n-1$ và $n-2$ đều được tô đỏ và $2n-3=n+n-3$ nên $n-3$ cũng được tô đỏ, tương tự, ta có tất cả các số từ $1$ đến $n-1$ được tô đỏ(vô lí)

Nếu $f_n=2n-5$, ta có cách tô như sau

Tô đỏ số $1$ và các số từ $\frac{n}{2}+1$ đến $n-1$

Tô xanh số $2n$ và các số từ $2$ đến $\frac{n}{2}$




#653520 Đề thi chọn đội tuyển chuyên Thái Bình

Đã gửi bởi JUV on 09-09-2016 - 23:03 trong Thi HSG cấp Tỉnh, Thành phố. Olympic 30-4. Đề thi và kiểm tra đội tuyển các cấp.

Bài 5: Gọi $S_{XYZ}$ là diện tích tam giác $XYZ$.Dễ thấy với $n=3,4$ thì thoả mãn đề bài. Ta sẽ chứng minh với $n=5$ thì không thoả mãn để bài. Giả sử $A_1,A_2,A_3,A_4,A_5$ là $5$ điểm thoả mãn đề bài, xét $T$ là $1$ bao lồi của $5$ điểm đó:

- Nếu $T$ là $1$ tam giác, giả sử là tam giác $A_1A_2A_3$ thì lúc đó $A_4,A_5$ nằm trong tam giác đó, do vậy $S_{A_1A_2A_4}+S_{A_1A_3A_4}+S_{A_3A_2A_4}=S_{A_1A_2A_5}+S_{A_1A_3A_5}+S_{A_3A_2A_5}=S_{A_1A_2A_3}$. Lại có $S_{A_1A_2A_5}-S_{A_1A_2A_4}=S_{A_1A_3A_5}-S_{A_1A_3A_4}=S_{A_3A_2A_5}-S_{A_3A_2A_4}=r_5-r_4$ nên $r_5=r_4$, hay $S_{A_1A_2A_5}-S_{A_1A_2A_4}=S_{A_1A_3A_5}-S_{A_1A_3A_4}=S_{A_3A_2A_5}-S_{A_3A_2A_4}=r_5-r_4=0$, nên $A_4A_5$ song song với cả 3 đoạn $A_1A_2,A_2A_3,A_3A_1$(vô lí)

-Nếu $T$ là $1$ tứ giác, giả sử là $A_1A_2A_3A_4$ thì lúc đó $A_5$ nằm trong $1$ trong $2$ tam giác $A_1A_2A_3$ hoặc $A_4A_3A_2$. Giả sử $A_5$ nằm ngoài $A_1A_2A_3$, lúc đó $A_4,A_5$ nằm cùng phía bờ $A_2A_3$, khác phía $A_1$. $2$ điểm đó cũng nằm cùng phía với $A_2$ bờ $A_1A_3$ và $A_3$ bờ $A_2A_1$ nên $S_{A_1A_2A_4}+S_{A_1A_3A_4}-S_{A_3A_2A_4}=S_{A_1A_2A_5}+S_{A_1A_3A_5}-S_{A_3A_2A_5}=S_{A_1A_2A_3}$. Lập luận tương tự cũng có $A_4A_5$ song song với cả $3$ cạnh tam giác $A_1A_2A_3$, vô lí.

-Nếu $T$ là ngũ giác $A_1A_2A_3A_4A_5$ ,xét vị trí các điểm $A_4,A_5$ với tam giác $A_1A_2A_3$ cũng có điều vô lí

Vậy $n=5$ không thoả mãn. Với $n>5$, ta chỉ cần xét $5$ điểm bất kì trong $n$ điểm đó để suy ra điều vô lí. 

Vậy $n=3$ hoặc $n=4$




#655970 Đề thi chọn đội tuyển HSG quốc gia tỉnh Đồng Nai năm học 2016-2017

Đã gửi bởi JUV on 29-09-2016 - 12:17 trong Thi HSG cấp Tỉnh, Thành phố. Olympic 30-4. Đề thi và kiểm tra đội tuyển các cấp.

Câu 7: Dễ thấy $2017$ là số nguyên tố. Nếu $n=1008$, ta có $1008$ số $1,2,...,1008$ không thoả mãn đề bài. Giả sử tồn tại $1009$ số không thoả mãn đề bài. Nếu trong $1009$ số đó có $1$ số $a_i$ chia hết cho $2007$, $1$ số $a_j$ không chia hết thì đặt $a_i=2017c$, có$\frac{a_i+a_j}{(a_i;a_j)}> \frac{2017c}{(2017c;a_j)}=\frac{2017c}{(c;a_j)}\geq \frac{2017c}{c}=2017$(vô lí). Nếu tất cả các số đều chia hết cho $2017$ thì ta chia các số đó cho $2017$, ta cũng được $1009$ số không thoả mãn đề bài. Cứ chia như vậy cho đến khi có $1$ số không chia hết cho $2017$. Nếu lúc đó còn $1$ số chia hết cho $2017$ thì lập luận tương tự suy ra điều vô lí. Nếu tất cả các số đều không chia hết cho $2017$, giả sử tồn tại $a_i;a_j$ sao cho $a_i-a_j=2017c$ với $c$ là $1$ số nguyên dương thì $\frac{a_i+a_j}{(a_i;a_j)}= \frac{2017c+2a_j}{(a_j;2017c)}> \frac{2017c}{(a_j;c)}\geq \frac{2017c}{c}=2017$(vô lí). Nếu không có $2$ số nào đồng dư với nhau $\pmod{2017}$ thì xét $1008$ cặp $(1;2016),(2;2015);...;(1008;1009)$, lúc đó tồn tại $2$ số $a_i;a_j$ trong $1009$ số sao cho $a_i+a_j=2017d$ với $d$ nguyên. Vì vậy $\frac{a_i+a_j}{(a_j;a_i)}= \frac{2017d}{(a_j;a_i+a_j)}= \frac{2017d}{(2017d;a_j)}=\frac{2017d}{(d;a_j)}\geq \frac{2017d}{d}= 2017$ (vô lí). Vậy giả sử sai, hay luôn tồn tại $2$ trong $1009$ số thoả mãn đề bài. Vậy $n=1009$




#654923 Đề thi chọn đội tuyển quốc gia THPT chuyên KHTN - ĐHQG Hà Nội vòng 1 năm 2016

Đã gửi bởi JUV on 20-09-2016 - 21:22 trong Thi HSG cấp Tỉnh, Thành phố. Olympic 30-4. Đề thi và kiểm tra đội tuyển các cấp.

Bài 2 ngày 2. Ý tưởng chính là quy nạp

Dễ thấy với $n=2$ thì ít hơn $2$ bước di chuyển nên bài toán đúng với $n=2$.

Giả sử đúng đến $n$, ta chứng minh đúng với $n+1$.

Xét $n+1$ ở vị trí $k$ với $k$ khác $n+1$.

Nhận thấy rằng ta không thể lấy một số ở vị trí trước $k$ rồi thêm vào sau $k$ trước khi đưa số $n+1$ vào vị trí cuối cùng nên khoảng cách từ vị trí của số $n+1$ đến vị trí cuối cùng chỉ có thể giảm hoặc không đổi.

Sau mỗi bước di chuyển, nếu một số ở sau vị trí $k$ nhỏ hơn hoặc bằng $k$ được chuyển về trước vị trí $k$ thì khoảng cách từ vị trí của số $n+1$ đến vị trí cuối cùng sẽ giảm xuống $1$. Do đó ta chỉ cần xét cho trường hợp xấu nhất là tất cả các số sau vị trí $k$ đều lớn hơn $k$.

Ta thấy tồn tại một song ánh giữa hai tập $\left \{ k+1,k+2,...,n+1 \right \}\to\left \{ 1,2,...,n-k+1 \right \}$ nên để tất cả các số sau vị trí $k$ về vị trí đúng thì cần ít hơn $2^{n-k+1}$ bước. Do đó để số $n+1$ về đúng vị trí của nó thì cần ít hơn hoặc bằng $2^{n-k+1}$.

Theo giả thiết quy nạp để chuyển dãy là hoán vị của $\left \{ 1,2,3,..,n \right \}$ về vị trí đúng thì cần ít hơn $2^n$ bước nên để chuyển dãy là hoán vị của $\left \{ 1,2,3,..,n+1 \right \}$ thì cần ít hơn $2^n+2^{n-k+1}\leq 2^n+2^n=2^{n+1}$ (bước).

Do đó ta có đpcm.

Đoạn trường hợp xấu nhất là tất cả các số sau vị trí $k$ lớn hơn $k$ có vấn đề rồi: Cho dù số $x$ nhỏ hơn $k$ được chọn và di chuyển về sau $n$ thì cũng đã tốn $1$ bước rồi, ví dụ cho số $1$ ở vị trí cuối thì khi di chuyển về vị trí đầu là cũng mất $1$ bước và còn làm xáo trộn trật tự của các số đứng đằng sau $k$ nên tất nhiên số bước để $n$ đến vị trí $n$ cũng thay đổi không rõ như thế nào dù khoảng cách $n$ cần phải di chuyển giảm, huống hồ những số nhỏ hơn $k$ lại phân bố không biết lối nào mà lần (không phải khoảng cách càng nhỏ thì số bước càng nhỏ)




#654926 Đề thi chọn đội tuyển quốc gia THPT chuyên KHTN - ĐHQG Hà Nội vòng 1 năm 2016

Đã gửi bởi JUV on 20-09-2016 - 21:30 trong Thi HSG cấp Tỉnh, Thành phố. Olympic 30-4. Đề thi và kiểm tra đội tuyển các cấp.

Bài 2: Ta sẽ chứng minh rằng với số $x$ bất kì với $0<x<n+1$ thì $x$ sẽ được đứng cuối nhiều nhất $2^{x-1}$ lần (Số lần đứng cuối của $x$ được tính bằng số bước mà sau mỗi bước đó, ta thu được $1$ hoán vị với số $x$ là số sẽ được chọn để chuyển về đúng vị trí trong bước sau). Với $x=1$ thì ta thấy ngay rằng $1$ chỉ đứng cuối nhiều nhất $1=2^0$ lần vì sau lần đầu tiên đứng cuối, nó sẽ chyển về vị trí thứ nhất và sẽ không di chuyển nữa. Với $x=2$ thì sau lần đứng cuối đầu tiên, số $2$ sẽ đi về vị trí thứ $2$. Nếu lúc đó trước số $2$ là số $1$ thì nó sẽ không di chuyển nữa, nếu không thì lúc đó số $1$ đứng đằng sau số $2$ nên trước khi số $2$ đứng cuối $1$ lần nữa thì số $1$ sẽ đứng cuối trước số $2$ $1$ lần và di chuyển về vị trí $1$. Sau đó số $2$ sẽ đứng cuối $1$ lần nữa và di chuyển về vị trí $2$ trước vị trí $1$ mà số $1$ đang đứng, tức số $2$ đứng cuối nhiều nhất $2=2^1$ lần. Mệnh đề đúng với $x=1,2$, giả sử mệnh đề đúng với $x=k<n$. Xét số $k+1$ trong dãy số, nếu nó không đứng cuối lần nào thì ta có ngay điều phải chứng minh. Xét lần đứng cuối đầu tiên của $k+1$ và nó di chuyển về vị trí $k+1$, ta thấy rằng để nó có thể thay đổi vị trí của mình một lần nữa thì cần phải đến $1$ lúc nào đó có $1$ số $x<k+1$ đứng cuối rồi di chuyển về vị trí $x$ trước $k+1$ làm vị trí của $k+1$ tăng thêm $1$. Nghĩa là sau lần đứng cuối đầu tiên thì để $k+1$ đứng cuối $1$ lần nữa thì cần có $1$ số $x<k+1$ đứng cuối trước nó. Mỗi số $1$ đến $k$ tổng cộng đứng cuối nhiều nhất $2^0+2^1+...+2^{k-1}=2^k-1$ lần theo quy nạp và trước khi để $k+1$ đứng cuối lần nữa thì cần phải có $1$ số trong $k$ số đó đứng cuối trước nên số lần $k+1$ đứng cuối sau lần thứ nhất nhiều nhất $2^k-1$ lần. Vậy $k+1$ đứng cuối nhiều nhất $2^k$ lần. Bước chứng minh quy nạp hoàn tất. Ta thấy số bước bằng tổng số lần các số $1,2,...,n$ đứng cuối nên số bước nhiều nhất là $2^0+2^1+...+2^{n-1}=2^n-1<2^n$ lần




#655001 Đề thi chọn đội tuyển quốc gia THPT chuyên KHTN - ĐHQG Hà Nội vòng 1 năm 2016

Đã gửi bởi JUV on 21-09-2016 - 16:48 trong Thi HSG cấp Tỉnh, Thành phố. Olympic 30-4. Đề thi và kiểm tra đội tuyển các cấp.

Bài 2 ngày 2: Sau mỗi bước sẽ có 1 tập vài quyển sách ở đúng vị trí, có tất cả $2^n$ tập, ta sẽ chứng minh không có 2 bước nào có 2 tập nào giống nhau

Quy ước lấy chiều từ trái sang phải

Ta thấy 1 quyển nếu chưa ở vị trí đúng thì sau khi bị di chuyển bởi quyển khác chỉ có thể sang phải, nếu đã ở vị trí đúng rồi thì sau đó không thế về vị trí nào trước vị trí đúng  

- Giả sử có 2 bước thứ $x$ và $y (x<y)$ có tập $a_1,...a_k$ trùng nhau

+ TH1: Nếu bước thứ $y$ chuyển quyển thứ $a_k$, vị trí đúng của $a_k$ không phải ở cuối

Các quyển ở sau $a_k$ sau bước thứ $x$ không thể có vị trí đúng là trước vị trí đúng của $a_k$ nếu không sẽ có 1 quyển được đưa về vị trí đúng trước bước thứ $y$, tạo ra 1 tập khác 

Vì các quyển sau $a_k$ có vị trí đúng sau $a_k$ nên sẽ di chuyển về vị trí đúng mà không ảnh hưởng tới các quyển khác, sau đó sẽ di chuyển quyển sai vị trí gần nhất trước $a_k$, nhưng lúc này không thể thay đổi vị trí $a_k$ và các quyển sau nữa.

+ TH2: bước thứ $y$ di chuyển quyển $a_i (i \neq k)$

Khi di chuyển $a_i$ nghĩa là tất cả các quyển sau $a_i$ đã đúng vị trí và khi di chuyển $a_i$ các quyển đó không đổi vị trí, suy ra các quyển đó đúng vị trí từ bước $x$, tuy nhiên khi điều đó có nghĩa tất cả các quyển từ $a_i$ trở đi đều không bị di chuyển, nên từ bước $x$ tới bước $y$ ta chỉ có thể di chuyển các quyển trước $a_i$, mâu thuẫn với việc quyển bị di chuyển là $a_i$

Đề bài cần chứng minh sau ít hơn $2^n$ lần chứ không phải nhiều nhất $2^n$ lần




#655372 Đề thi chọn đội tuyển quốc gia THPT chuyên KHTN - ĐHQG Hà Nội vòng 2 năm 2016

Đã gửi bởi JUV on 24-09-2016 - 18:04 trong Thi HSG cấp Tỉnh, Thành phố. Olympic 30-4. Đề thi và kiểm tra đội tuyển các cấp.

Câu 4: Xét $11$ số $a+1,...,a+11 (a<2006)$. Chọn ra các số sao cho không có $2$ số nào có hiệu là $4$ hoặc $7$. Chia tập $D_a= \left \{ a+1;...;a+11 \right \}$ thành các nhóm $(a+1,a+8),(a+2,a+6),(a+3,a+7),(a+4,a+11),(a+5,a+9),(a+10)$.Không có $2$ số được chọn nào được thuộc cùng một nhóm và không thể chọn số $a+10$ rồi chọn mỗi nhóm $1$ số nên có nhiều nhất $5$ số được chọn . Chia tập $2016$ số nguyên dương đầu tiên thành các tập $(2016;2015;2014),A_0,A_{11},...,A_{2002}$, tâp đầu chọn nhiều nhất $3$ số, các tập tiếp theo, mỗi tập chọn được nhiều nhất $5$ số, lưu ý là nếu cả $3$ số $2013,2014,2015$ được chọn thì chỉ chọn được nhiều nhất $4$ số ở $A_{2002}$ nên tổng số cách chọn nhiều nhất là $183\times 5+2=917$, chẳng hạn khi ta chọn các số đồng dư $1,2,4,7,10 \pmod{11}$




#655702 Đề thi chọn đội tuyển quốc gia THPT chuyên KHTN - ĐHQG Hà Nội vòng 2 năm 2016

Đã gửi bởi JUV on 26-09-2016 - 22:49 trong Thi HSG cấp Tỉnh, Thành phố. Olympic 30-4. Đề thi và kiểm tra đội tuyển các cấp.

Bài 2: Ta nhận xét rằng để biểu diễn số $n!-1$ thì cần ít nhất $\frac{n(n-1)}{2}$ lá bài. Thật vậy thì ta có thể biểu diễn $n!-1=(n-1)((n-1)!)+(n-2)((n-2)!)+...+2.2!+1.1!$. Xét cách biểu diễn $n!-1=a_1.1!+a_2.2!+...+a_{n-1}.(n-1)!$ trong đó ta dùng lần lượt $a_1,a_2,...,a_{n-1}$ quân bài $1!,2!,...,(n-1)!$ và $a_1+a_2+...a_{n-1}$ nhỏ nhất. Ta thấy rằng $a_i<i+1$$\forall 0<i<n$, nếu không thì ta chỉ cần thay $i+1$ lá bài $i!$ thành $1$ lá bài $(i+1)!$ thì sẽ tạo được $1$ cách chọn bài ít hơn. Cộng số các lá bài $a_i$ lại ta có số lá bài dùng để biểu diễn $n!-1$ ít nhất là $\frac{n(n-1)}{2}$ lá bài. Dễ thấy là để biểu diễn $n!-1$ dưới ít nhất $\frac{n(n-1)}{2}$ lá bài thì các lá bài đó chỉ có thể là $i$ lá bài $i!$ $\forall 0<i<n$, tuy nhiên các lá bài đó không thể biểu diễn được $n!$ nên ta sẽ cần $\frac{n(n-1)}{2}+1$ lá bài để có thể thoả mãn đề bài (có thể thêm $1$ lá bài $1!$ hoặc $n!$). Vậy cần ít nhất $\frac{n(n-1)}{2}+1$ lá bài